If a wildlife preservation grant and a youth services grant are awarded in the same quarter of a particular calendar ...

EmilyMarieMenendez on September 5, 2018

Answer Choice A

I am a bit confused as to how answer choice A could be true. In the hypothetical, it is written out as: 1Q: 2Q:WM 3Q: 4Q:WY However, I was confused about the other M placement. If this was the hypothetical proving true that M could be awarded in Q2, wouldn't the other M have to be awarded in Q4 setting up a 0-2-1-3 distribution? Thanks for your help!

Replies
Create a free account to read and take part in forum discussions.

Already have an account? log in

Max-Youngquist on September 6, 2018

@emilyMarieMenendez in your diagram, you put Y in 4Q, but it could actually also be in 2Q as follows:

1Q: T
2Q: WMY
3Q: T
4Q: M

I hope that helps!

EmilyMarieMenendez on September 6, 2018

Oh I see, thanks!